-
Notifications
You must be signed in to change notification settings - Fork 38
/
Copy path10.tex
1397 lines (1180 loc) · 51.7 KB
/
10.tex
1
2
3
4
5
6
7
8
9
10
11
12
13
14
15
16
17
18
19
20
21
22
23
24
25
26
27
28
29
30
31
32
33
34
35
36
37
38
39
40
41
42
43
44
45
46
47
48
49
50
51
52
53
54
55
56
57
58
59
60
61
62
63
64
65
66
67
68
69
70
71
72
73
74
75
76
77
78
79
80
81
82
83
84
85
86
87
88
89
90
91
92
93
94
95
96
97
98
99
100
101
102
103
104
105
106
107
108
109
110
111
112
113
114
115
116
117
118
119
120
121
122
123
124
125
126
127
128
129
130
131
132
133
134
135
136
137
138
139
140
141
142
143
144
145
146
147
148
149
150
151
152
153
154
155
156
157
158
159
160
161
162
163
164
165
166
167
168
169
170
171
172
173
174
175
176
177
178
179
180
181
182
183
184
185
186
187
188
189
190
191
192
193
194
195
196
197
198
199
200
201
202
203
204
205
206
207
208
209
210
211
212
213
214
215
216
217
218
219
220
221
222
223
224
225
226
227
228
229
230
231
232
233
234
235
236
237
238
239
240
241
242
243
244
245
246
247
248
249
250
251
252
253
254
255
256
257
258
259
260
261
262
263
264
265
266
267
268
269
270
271
272
273
274
275
276
277
278
279
280
281
282
283
284
285
286
287
288
289
290
291
292
293
294
295
296
297
298
299
300
301
302
303
304
305
306
307
308
309
310
311
312
313
314
315
316
317
318
319
320
321
322
323
324
325
326
327
328
329
330
331
332
333
334
335
336
337
338
339
340
341
342
343
344
345
346
347
348
349
350
351
352
353
354
355
356
357
358
359
360
361
362
363
364
365
366
367
368
369
370
371
372
373
374
375
376
377
378
379
380
381
382
383
384
385
386
387
388
389
390
391
392
393
394
395
396
397
398
399
400
401
402
403
404
405
406
407
408
409
410
411
412
413
414
415
416
417
418
419
420
421
422
423
424
425
426
427
428
429
430
431
432
433
434
435
436
437
438
439
440
441
442
443
444
445
446
447
448
449
450
451
452
453
454
455
456
457
458
459
460
461
462
463
464
465
466
467
468
469
470
471
472
473
474
475
476
477
478
479
480
481
482
483
484
485
486
487
488
489
490
491
492
493
494
495
496
497
498
499
500
501
502
503
504
505
506
507
508
509
510
511
512
513
514
515
516
517
518
519
520
521
522
523
524
525
526
527
528
529
530
531
532
533
534
535
536
537
538
539
540
541
542
543
544
545
546
547
548
549
550
551
552
553
554
555
556
557
558
559
560
561
562
563
564
565
566
567
568
569
570
571
572
573
574
575
576
577
578
579
580
581
582
583
584
585
586
587
588
589
590
591
592
593
594
595
596
597
598
599
600
601
602
603
604
605
606
607
608
609
610
611
612
613
614
615
616
617
618
619
620
621
622
623
624
625
626
627
628
629
630
631
632
633
634
635
636
637
638
639
640
641
642
643
644
645
646
647
648
649
650
651
652
653
654
655
656
657
658
659
660
661
662
663
664
665
666
667
668
669
670
671
672
673
674
675
676
677
678
679
680
681
682
683
684
685
686
687
688
689
690
691
692
693
694
695
696
697
698
699
700
701
702
703
704
705
706
707
708
709
710
711
712
713
714
715
716
717
718
719
720
721
722
723
724
725
726
727
728
729
730
731
732
733
734
735
736
737
738
739
740
741
742
743
744
745
746
747
748
749
750
751
752
753
754
755
756
757
758
759
760
761
762
763
764
765
766
767
768
769
770
771
772
773
774
775
776
777
778
779
780
781
782
783
784
785
786
787
788
789
790
791
792
793
794
795
796
797
798
799
800
801
802
803
804
805
806
807
808
809
810
811
812
813
814
815
816
817
818
819
820
821
822
823
824
825
826
827
828
829
830
831
832
833
834
835
836
837
838
839
840
841
842
843
844
845
846
847
848
849
850
851
852
853
854
855
856
857
858
859
860
861
862
863
864
865
866
867
868
869
870
871
872
873
874
875
876
877
878
879
880
881
882
883
884
885
886
887
888
889
890
891
892
893
894
895
896
897
898
899
900
901
902
903
904
905
906
907
908
909
910
911
912
913
914
915
916
917
918
919
920
921
922
923
924
925
926
927
928
929
930
931
932
933
934
935
936
937
938
939
940
941
942
943
944
945
946
947
948
949
950
951
952
953
954
955
956
957
958
959
960
961
962
963
964
965
966
967
968
969
970
971
972
973
974
975
976
977
978
979
980
981
982
983
984
985
986
987
988
989
990
991
992
993
994
995
996
997
998
999
1000
\chapter{指数函数与对数函数}
\section{有理指数函数}
在本教材第三册中,已经把指数幂的定义范围从正整指
数逐步推广到“负整数”,“正、负分数”,在逐步推广过程
中,我们始终遵守的指导原则是保有指数法则:
\[a^m\cdot a^n=a^{m+n},\qquad (a^m)^n=a^{m\cdot n}\]
指数在有理数系$\mathbb{Q}$内,我们有下面的指数幂的定义:
\[\begin{split}
a^n=\underbrace{a\cdot a\cdot a\cdots a}_{\text{$n$个$a$}},&\qquad (n\in\mathbb{N})\\
a^0=1,&\qquad (a\ne 0)\\
a^{-n}=\frac{1}{a^n},&\qquad (a\ne 0)\\
a^{\tfrac{m}{n}}=\sqrt[n]{a^m}=\left(\sqrt[n]{a}\right)^m,&\qquad (a\ge 0,\; m,n\in\mathbb{N})\\
a^{-\tfrac{m}{n}}=\frac{1}{a^{\tfrac{m}{n}}},&\qquad (a> 0)\\
\end{split}\]
采用上面定义后,我们在第三册中也证明了正实数$a$和$b$的有
理指数幂依然满足指数运算法则:
\[a{\alpha}\cdot a^{\beta}=a^{\alpha+\beta},\qquad (a^{\alpha})^{\beta}=a^{\alpha\beta},\qquad (a\cdot b)^{\alpha}=a^{\alpha}\cdot b^{\alpha}\]
这里$\alpha,\beta \in \mathbb{Q}$.
这样一来,函数$a^x\; (a>0)$对于任意有理数$x$都有定义
了.我们称它为有理指数函数,这个函数具有上面所说的三
个性质.下面将进一步研讨这个函数的其它重要性质:
\begin{blk}{性质1}
\begin{enumerate}
\item 若$a>1$, 当有理数$x>0$时,则$a^x>1$, 当有理
数$x<0$时,则$a^x<1$.
\item 若当$0<a<1$, 有理数$x>0$时,则$a^x<1$, 当有理数$x<0$
时,则$a^x>1$.
\end{enumerate}
\end{blk}
\begin{proof}
\begin{enumerate}
\item 若$a>1$,
\begin{enumerate}
\item 设$x=\frac{m}{n}>0,\; (m,n\in\mathbb{N})$, 则$a^x=
a^{\tfrac{m}{n}}=\sqrt[n]{a^m}$, 因为$a>1$, 所以$a^m>1$ (幂函数$f(x)=x^m$在
$[0,+\infty)$上是严格递增的),又$\sqrt[n]{a^m}>1$ (幂函数$f(x)=x^{\tfrac{1}{n}}$
在$[0,+\infty)$上是严格递增的),即$a^x>1$.
\item 设$x<0$,
$x=-x_1,\; (x_1>0)$,则$0<a^x=a^{-x_1}=\frac{1}{a^{x_1}}<1$,($\because\; a^{x_1}>1$).
\end{enumerate}
\item 若$0<a<1$,
\begin{enumerate}
\item 设$a=\frac{1}{a_1},\; a_1>1$, 则当$x>0$, $a^x=\left(\frac{1}{a_1}\right)^x=\frac{1}{a^x_1}<1$, ($\because\; a_1^{x}>1$).
\item 设$x<0$, $x=-x_1,\; (x_1>0)$则
$a^x=a^{-x_1}=\frac{1}{a^{x_1}}>1$, ($\because\; a^{x_1}<1$).
\end{enumerate}
\end{enumerate}
\end{proof}
性质1的几何意义表明:当$a>1$时,有理指数函数$y=
a^x$的图象上的点在有单斜线的区域I和II的部分;当$0<a<
1$时,$y=a^x$的图象上的点在有双斜线的区域III和IV的部分
(图6.1).
\begin{figure}[htp]
\centering
\begin{tikzpicture}[>=latex, scale=.7]
\draw[->] (-3,0)--(4,0)node[right]{$x$};
\draw[->] (0,-1)--(0,5)node[right]{$y$};
\node at (-.25,-.25){$O$};
\draw[very thick](-3,1)--(3.5,1)node[right]{$y=1$};
\fill[pattern = north east lines] (-3,1) rectangle (0,0);
\fill[pattern = north east lines] (3,4.5) rectangle (0,0);
\fill[pattern = crosshatch] (-3,4.5) rectangle (0,1);
\fill[pattern = crosshatch] (3,1) rectangle (0,0);
\end{tikzpicture}
\caption{}
\end{figure}
\begin{blk}{性质2}
\begin{enumerate}
\item 若$a>1$, $x_1<x_2$,则$a^{x_1}<a^{x_2}$, 即底数大于1的
有理指数函数$a^x$是递增的;
\item 若$0<a<1$, $x_1<x_2$,则$a^{x_1}>a^{x_2}$,即底数小于1的正数的有理指数函数$a^x$是递减的.
\end{enumerate}
\end{blk}
\begin{proof}
若$a>1$和$x_1<x_2$, 那么
\[a^{x_2}-a^{x_1}=a^{x_1}\left(\frac{a^{x_2}}{a^{x_1}}-1\right)=a^{x_1}\left(a^{x_2-x_1}-1\right)\]
因为$x_2-x_1>0$, $a>1$, 所以$a^{x_2-x_1}>1$, 又$a^{x_1}>0$. 因
此,$a^{x_2-x_1}>0$, 即$f(x)=a^x,\; (a>1)$是递增的.
若$0<a<1$和$x_1<x_2$, 那么
\[a^{x_2}-a^{x_1}=a^{x_1}\left(a^{x_2-x_1}-1\right)\]
因为$x_2-x_1>0$, $0<a<1$, 所以$a^{x_2-x_1}<1$, 又$a^{x_1}>0$, 因
此$a^{x_2-x_1}<0$, 即$f(x)=a^x,\; (0<a<1)$是递减的.
\end{proof}
我们现在的任务是要把有理指数函数开拓为一个定义在
实数集上的连续函数.能否做到这一点的关键是如何对全体
无理点补充定义,使得指数函数在整个实数轴$\mathbb{R}$上处处连
续.为此,我们先说明有理指数函数的一个极限性质.
\begin{blk}{性质3}
设$a>0$,则当$n\to +\infty$时,数列$\left\{a^{\tfrac{1}{n}}\right\}$的极限是1,即
\[\lim_{n\to\infty}a^{\tfrac{1}{n}}=1\]
\end{blk}
\begin{proof}
\begin{enumerate}
\item 当$a=1$时,结论自然成立.
\item 当$a>1$时,因为
$\frac{1}{n}>0$, 所以$a^{\tfrac{1}{n}}>1$ (性质1),
设$a^{\tfrac{1}{n}}=1+h$, 其中$h>0$, 两边$n$次方,得到
\[ a=(1+h)^n\]
由贝努力不等式得
\[ a=(1+h)^n>1+nh\]
所以,
$ 0<h<\frac{a-1}{n},\qquad 1<1+h<1+\frac{a-1}{n}$,
即:
\[1<a^{\tfrac{1}{n}}<1+\frac{a-1}{n}\]
再令$n\to +\infty$, 由上式就得到
\[1\le \lim_{n\to\infty}a^{\tfrac{1}{n}}\le 1 \]
因此\[\lim_{n\to\infty}a^{\tfrac{1}{n}}=1\]
\item 当$0<a<1$时,令$a=\frac{1}{b}$, 则$b>1$, 由上面的证明
得到\[\lim_{n\to\infty}b^{\tfrac{1}{n}}=1\]
于是
\[\begin{split}
\lim_{n\to\infty}a^{\tfrac{1}{n}}&=\lim_{n\to\infty}\left(\frac{1}{b}\right)^{\tfrac{1}{n}}=\lim_{n\to\infty}\frac{1}{b^{\tfrac{1}{n}}}\\
&=\frac{1}{\displaystyle\lim_{n\to\infty}b^{\tfrac{1}{n}}}=\frac{1}{1}=1
\end{split}\]
\end{enumerate}
\end{proof}
性质3可以进一步推广到下面的推论:
\begin{blk}{推论}
若$a>0$且$a\ne 1$, 有理数数列$\{h_i\},\; i=1,2,3,\ldots$,
以0为极限,即$\Lim_{i\to\infty}h_i=0$, 那么
\[\lim_{i\to\infty}a^{h_i}=1\]
\end{blk}
\begin{proof}
先设$a>1$, 因为$\Lim_{i\to\infty}h_i=0$, 必定存在这样的
自然数$N$, 使得当$i\ge N$时,$|h_i|<1$, 从而$\frac{1}{|h_i|}>1$.
用$m_i$表示$\left[\frac{1}{|h_i|}\right]$,
即不大于$\frac{1}{|h_i|}$
的最大整数,于是
\begin{equation}
m_i=\left[\frac{1}{|h_i|}\right]\le \frac{1}{|h_i|}<m_i+1
\end{equation}
所以,当$i\ge N$时,有
\[\frac{1}{m_i+1}<|h_i|\le\frac{1}{m_i}\]
由$h\to 0$知,$\frac{1}{|h_i|}\to \infty$. 从而由$m_i+1>\frac{1}{|h_i|}$知,$m_i\to\infty$.根据有理指数幂的单调性,得
\[1<a^{|h_i|}<a^{\tfrac{1}{m_i}},\qquad (a>1)\]
仿照性质3的证明,令$b_i=a^{\tfrac{1}{m_i}}-1>0$, 于是,
\[\begin{split}
a^{\tfrac{1}{m_i}}&=(1+b_i)\\
a&=(1+b_i)^{m_i}>1+m_ib_i\\
& 0<b_i<\frac{a-1}{m_i}
\end{split}\]
当$i\to 0$时,$m_i\to \infty$,
$\therefore\quad b_i\to 0$, 即$a^{h_i}\to 1$, 从而当$i\to \infty$时,$|h_i|\to 0$, $a^{|h_i|}\to 1$, 即$a^{h_i}\to 1$.
若$0<a<1$, 令$b=\frac{1}{a}>1$, 于是
\[\lim_{i\to \infty} a^{h_i}=\lim_{i\to \infty} \left(\frac{1}{b}\right)^{h_i}=\frac{1}{\Lim_{i\to \infty} b^{h_i}}=\frac{1}{1}=1\]
\end{proof}
应用这个推论,我们可以说明当有理数$x$的变化够小时,
有理指数函数$f(x)=a^x$的变化可以任意小.
\begin{blk}{性质4}
当指数x的变化够小时,有理指数函数$f(x)=
a^x$的变化可以任意小.
\end{blk}
\begin{proof}
设指数$x$从有理数$x_1$变化到有理数$x_2=x_1+h_i$,($h_i$
是有理数),且当$(x_2-x_1)\to 0$时,数列$\{h_i\}$以0为极限,于
是
\[\begin{split}
\lim_{x_2\to x_1}\left(a^{x_2}-a^{x_1}\right)&=\lim_{i\to \infty}\left(a^{x_1+h_i}-a^{x_1}\right)\\
&=a^{x_1}\cdot \lim_{i\to \infty}\left(a^{x_i}-1\right)=0
\end{split}\]
这就是说,只要$|h_i|$够小,那么$|a^{x_2}-a^{x_1}|$
就小于任意给定的正数$\varepsilon$.
\end{proof}
综合有理指数函数的性质,我们可以想象出$y=a^x\; (a>
1)$的图象如图10.2所示,但是我们不能用一条连续不断的
曲线把它画出来,因为指数$x$取无理数时,$a^x$还没有意义,
因而在有理指数函数的图象上,处处有空隙.下一节将由有理
指数函数的单调性和性质4, 适当给无理指数幂补充定义使
得指数函数在$\mathbb{R}$上处处连续.
\begin{figure}[htp]
\centering
\begin{tikzpicture}[>=latex, scale=.7]
\draw[->] (-2,0)--(5,0)node[right]{$x$};
\draw[->] (0,-1)--(0,5)node[right]{$y$};
\node at (-.35,-.35){$O$};
\draw[dashed] (-2,1)--(4.5,1)node[right]{$y=1$};
\draw[domain=-2:3.5, samples=30, very thick, dashed]plot(\x, {1.6^(\x)});
\node at (0,1.3)[left]{$(0,1)$};
\node at (3,1.6^3)[right]{$y=a^x,\quad (a>1)$};
\end{tikzpicture}
\caption{}
\end{figure}
\section*{习题10.1}
\addcontentsline{toc}{subsection}{习题10.1}
\begin{enumerate}
\item 计算下列各式的值:
\begin{enumerate}
\item $25^{3 / 2} \cdot 8^{4 / 3}$
\item $(0.09)^{1 / 2}+64^{2 / 3}+0.125^{2 / 3}-\frac{1}{16^{-3 / 2}}$
\item $64^{1.5} \cdot(32)^{0.4} \div\left(\frac{9}{25}\right)^{-3 / 2}$
\item $\left(\frac{81}{16}\right)^{-0.25}\left(5^{2}-0.1^{2} \cdot\left(\frac{1}{4}\right)^{-3}\right)^{2}$
\item $\left[\frac{3}{9}-\left(\frac{2}{3}\right)^{-1}\right]^{-1}$
\item $(\sqrt{2})^{1.5}+\left(11+\frac{\sqrt[5]{5}}{5^{-0.8}}\right)^{-1 / 4}$
\item $\left[\left(\frac{3}{4}\right)^{0}\right]^{-0.5}-7.5(\sqrt{4})^{2}-(-2)^{-4}+81^{0.25}$
\item $\left[\frac{1}{4}\left(0.027^{2 / 3}+15 \times 0.0016^{3 / 4}+1\right)\right]^{-1 / 2}$
\item $6\left[\sqrt{3}\left(\sqrt{3}+2 \sqrt{2}+\frac{2}{3^{1 / 2}}\right)\right] \times\left(3^{1 / 2}+2^{1 / 2}\right)^{-2} \times\left(3^{-1}+2^{-1}\right)$
\item 若 $a=(2+\sqrt{3})^{-1},\quad b=(2-\sqrt{3})^{-1}$, 计算 $(a+1)^{-1}+(b+1)^{-1}$
\end{enumerate}
\item 化简下列各式:
\begin{enumerate}
\begin{multicols}{2}
\item $b^{1 / 2} b^{1 / 3}$
\item $b^{1 / 2} b^{-1 / 3}$
\item $b^{-2 / 3} b^{3 / 5} ;$
\item $b^{-2 / 3} b^{3 / 5} ;$
\item $\sqrt{a} \cdot \sqrt[3]{a} \cdot \sqrt[5]{a}$
\item $\left[1-\left(a^{-1} b^{-1}\right)^{-1}\right]^{-2}$
\end{multicols}
\item $\left[a^{-1 / 2} b^{-1 / 2}+a^{-1 / 6}\left(b^{-5 / 6}-a^{-1 / 3} b^{-1 / 2}\right)\right]^{-3 / 2}$
\item $\frac{\left(a^{-1}+b^{-1}\right)(a+b)^{-1}}{\sqrt[6]{a^{4} \sqrt[5]{a^{-2}}}}$
\item $\frac{a^{2}+a^{-2}-2}{a^{2}-a^{-2}}$
\item $\left(a^{3 / 4}+b^{3 / 4}\right)\left(a^{3 / 4}-b^{3 / 4}\right) /\left(a^{1 / 2}-b^{1 / 2}\right)$
\item $\left(e^{3 / 2}+2+e^{-3 / 2}\right)\left(e^{3 / 2}-2+e^{-3 / 2}\right)$
\item $\left(a^{1 / 3}+a^{-1 / 3}\right)\left(a^{2 / 3}-1+a^{2 / 3}\right)$
\item $\frac{m-n}{m^{1 / 2}-n^{1 / 2}}+\frac{m^{3 / 2}+n^{3/2}}{m^{1 / 2}+n^{1 / 2}}$
\item $\frac{x^{2 p(q+1)}-y^{2 q(p-1)}}{x^{p(q+1)}-y^{q(p-1)}}$
\item $\left(a^{4 / 3}-2+a^{-4 / 3}\right)\left(a^{2 / 3}-a^{-2 / 3}\right)$
\item $\frac{m-n}{m^{1 / 2}-n^{1 / 2}}+\frac{m^{3 / 2}+n^{3/2}}{m^{1 / 2}+n^{1 / 2}}$
\item $\left[\frac{4 a-9 a^{-1}}{2 a^{1 / 2}-3 a^{-1 / 2}}+\frac{a-4+3 a^{-1}}{a^{1 / 2}-a^{-1 / 2}}\right]^{2}$
\end{enumerate}
\item 解下列各方程:
\begin{enumerate}
\begin{multicols}{2}
\item $\sqrt{2 x-3}=4-x$
\item $\sqrt{2 x+8}+\sqrt{x+5}=7$
\item $x^{-1 / 4}+x^{-1 / 2}-6=0$
\item $x^{1 / 2}+x^{-1 / 2}-\frac{10}{3}=0$
\end{multicols}
\item $\sqrt[n]{(x+1)^{2}}+\sqrt[n]{(x-1)^{2}}=4 \sqrt[n]{x^{2}-1}$
\end{enumerate}
\item 设 $h_{i}=\frac{100}{2 i+1},\quad m_{i}=\left[\frac{1}{h_{i}}\right]=\left[\frac{2 i+1}{100}\right]$
\begin{enumerate}
\item 求证数列$\{h_i\}=\left\{\frac{100}{2 i+1}\right\}$递减,并求使$h_i=\frac{100}{2i+1}<1$的$i$的范围;
\item 当$i=10,49,50,100,1000$时,求$m_i$的值;
\item 求证当$i\ge 50$时,不等式$1<100^{\tfrac{100}{2i+1}}<100^{\tfrac{1}{m_i}}$成立;
\item 求证:$\Lim_{i\to\infty}\left(100^{\tfrac{1}{m_i}}-1\right)=0,\quad \Lim_{i\to\infty}100^{h_i}=1$.
\end{enumerate}
\end{enumerate}
\section{无理指数幂的定义}
要把指数幂的定义由有理数推广到实数,自然又得用逼
近法.
设$\beta$是一个无理数,我们可以用两个有理数列$\{r_n\}$, $\{s_n\}$
去左、右夹逼,即$r_n\to \beta\leftarrow s_n$, 从而$\Lim_{n\to\infty}r_n=\Lim_{n\to\infty}s_n=\beta$. 现在
的问题是数列$\{a^{r_n}\}$,$\{a^{s_n}\}$,(这里$a>0$)的极限是否存
在?如果存在的话,我们就可以定义
\[a^{\beta}=\Lim_{n\to\infty}a^{r_n}=\Lim_{n\to\infty}a^{s_n}\]
从而就可以把有理指数函数$a^x$开拓为在$\beta$点连续的函数:
\[a^x\; (a>0,\; x\in \mathbb{Q}\cup\{\beta\})=\begin{cases}
a^x\; (x\in\mathbb{Q})\\
a^{\beta}=\Lim_{n\to\infty}a^{r_n}=\Lim_{n\to\infty}a^{s_n}
\end{cases}\]
\begin{blk}{引理}
设 $r_{n} \rightarrow \beta \leftarrow s_{n}$, 则
\begin{enumerate}
\item 当$a>1$时,$a^{r_1}\le a^{r_2}\le \cdots\le a^{r_n}\le \cdots \le a^{s_n}\le \cdots\le a^{s_2}\le a^{s_1}$,且$\left(a^{r_n}-a^{s_n}\right)\to 0$
当$0<a<1$时,$a^{r_1}\ge a^{r_2}\ge \cdots\ge a^{r_n}\ge \cdots \ge a^{s_n}\ge \cdots\ge a^{s_2}\ge a^{s_1}$,且$\left(a^{r_n}-a^{s_n}\right)\to 0$
\item $\lim a^{r_n}=\lim a^{s_{n}}=A$ (即极限存在)
\end{enumerate}
\end{blk}
\begin{proof}
$a>1$和$0<a<1$这两种情形是完全相似的,只是
不等式方向反过来罢了,所以下面只讨论$a>1$的情形,
($a=1$时它的任何方幂都是1, 所以$1^{\beta}=1$).我们只需
证明下述两点:
\begin{enumerate}
\item $a>1$, $s>r$时,则$a^s>a^r$,(性质2).
\item $\because\quad $当$n\to\infty$时,$s_n-r_n=h_n\to 0$,
$\therefore\quad $由性质4得
\[ a^{s_n}-a^{r_n}\to 0\]
由实数完备性,存在一个唯一实数
\[ A=\lim a^{s_n}=\lim a^{r_n}\]
\end{enumerate}
\end{proof}
\begin{blk}{定义}
设$\beta$是一任意无理数,$r_n\to\beta\leftarrow s_n$是$\beta$的左、右夹
逼数列,并且$u>0$, 则定义
\[a^{\beta}=\lim a^{r_n}=\lim a^{s_n}\]
\end{blk}
我们要说明这个定义的合理性,即上述定义和$\beta$的夹逼有
理数列的选取无关.
设$r'_n\to\beta\leftarrow s'_n$是另外一对夹逼数列,则
\[r'_n\to\beta \leftarrow s_n,\qquad r_n\to\beta\leftarrow s'_n\]
由上述引理就有
\[\lim a^{r'_n}=\lim a^{s_n}=\lim a^{r_n}=\lim a^{s'_n}\]
在实数轴$\mathbb{R}$上,对每一个无理点,都补充这样的定义,
于是我们就把有理指数函数开拓为一个在实数轴上处处有定
义的指数函数$a^x,\; (a>0,\; x\in\mathbb{R})$.
下面我们将证明这样定义的无理指数幂仍满足指数
法则.
\begin{blk}{定理}
指数法则$a^{\beta}\cdot a^{\gamma}=a^{\beta+\gamma}$, $\left(a^{\beta}\right)^{\gamma}=a^{\beta\cdot \gamma}$, $(ab)^{\beta}=a^{\beta}\cdot b^{\beta}$对于任何实数$\beta$, $\gamma$都成立.
\end{blk}
\begin{proof}
当$\beta$, $\gamma$是有理数时,上述等式已在本教材第三册
第一章给出证明,所以我们只要说明$\beta$, $\gamma$是无理数的情形.
设$r_n\to\beta\leftarrow s_n$, $c_n\to\gamma\leftarrow d_n$分别是$\beta$, $\gamma$的左、右夹逼数列,于是
\[(r_n+c_n)\to \beta+\gamma \leftarrow (s_n+d_n)\]
\[\begin{split}
a^{\beta+\gamma}&=\lim a^{r_n+c_n}=\lim a^{r_n}\cdot a^{c_n}\\
&=\lim a^{r_n}\cdot \lim a^{c_n}=a^{\beta}a^{\gamma}
\end{split}\]
现在让我们来证明$(a^{\beta})^{\gamma}=a^{\beta\cdot \gamma}$(为了讨论的方便,我们
只讨论$a>1,\; \beta ,\gamma>0$的情形),
设$r_n\to \beta \leftarrow s_n,\quad c_n\to \gamma\leftarrow d_n$,$r_n, s_n,c_n,d_n>0$, 则有
\[r_n\cdot c_n\to \beta_{\gamma}\leftarrow s_n\cdot d_n\]
根据正分指数的幂函数与有理指数函数的单调性有
\[(a^{r_n})^{c_n}<(a^{\beta})^{c_n}<(a^{\beta})^{\gamma}<(a^{\beta})^{d_n}<(a^{s_n})^{d_n}\]
所以由有理指数法则,得到
\[a^{r_n\cdot c_n}=(a^{\beta})^{c_n}<(a^{\beta})^{\gamma}<(a^{s_n})^{d_n}=a^{s_n\cdot d_n}\]
\[60+(u,.u,p-pus)\]
$\therefore\quad $由引理知,存在唯一的极限
\[(a^{\beta})^{\gamma}=\lim a^{r_n\cdot c_n} =\lim a^{s_n\cdot d_n} =a^{\beta\cdot \gamma}\]
最后证明:$(ab)^{\beta}=a^{\beta}\cdot b^{\beta}$, 只讨论$a>1$, $b>1$的情形.
$\because\quad a>1,b>1$
$\therefore\quad ab>1$, 于是
\[a^{r_n}b^{r_n}=(ab)^{r_n}<(ab)^{\beta}<(ab)^{s_n}=a^{s_n}b^{s_n}\]
\[(ab)^{s_n}-(ab)^{r_n}\to 0\]
因此,$(ab)^{\beta}=\lim a^{r_n}b^{r_n} = \lim a^{r_n}\cdot \lim b^{r_n}=a^{\beta}\cdot b^{\beta}$
\end{proof}
\begin{example}
\begin{enumerate}
\item $10^{\sqrt{2}}\cdot 10^{\sqrt{3}}=10^{\sqrt{2}+\sqrt{3}}$
\item $\left[\left(\sqrt[3]{2}\right)^{\sqrt{8}}\right]^{\tfrac{\sqrt{2}}{2}}=2^{\tfrac{1}{3}\x 2\sqrt{2}\x\tfrac{\sqrt{2}}{2}}=2^{\tfrac{2}{3}}=\sqrt[3]{4}$
\item $\left(5^{-\sqrt{2}}a^{\sqrt{8}}b^{\tfrac{\sqrt{2}}{2}}\right)^{\tfrac{\sqrt{2}}{2}}=5^{-1}a^{2}b^{\tfrac{1}{2}} =\frac{a^2\sqrt{b}}{5} $
\end{enumerate}
\end{example}
\section{实指数函数}
总结上节推广的结果,就得到一个对任意实数$x$都有定
义的\textbf{实指数函数}:
\[f:\mathbb{R}\mapsto \mathbb{R}^+,\qquad \text{这里} f(x)=a^x,\; (a>0, \; \text{且}a\ne 1)\]
这个函数就叫做\textbf{以$a$为底的指数函数}.上节还说明了指
数函数是一个满足下面两个性质的连续函数:
\begin{enumerate}
\item $f(x_1+x_2)=a^{x_1+x_2}=a^{x_1}\cdot a^{x_2}=f(x_1)\cdot f(x_2)$
\item $f(kx)=a^{kx}=(a^x)^k=[f(x)]^k$
\end{enumerate}
现在,我们还须验证实指数幂保有有理指数幂的一切性
质,并且实指数函数是连续的.
\begin{blk}{性质1}
\begin{enumerate}
\item 若$a>1$, 当$x>0$时,则$a^x>1$;
当$x<0$时,则$a^x<1$.
\item 若$0<a<1$, 当$x>0$时,则$a^x<1$;
当$x<0$时,则$a^x>1$.
\end{enumerate}
\end{blk}
\begin{proof}
如果$x$是有理数,我们在第一节中给过证明,这里不
再重述,所以我们只要证明$x$是无理数的情形,设$x>0$,
且$c_n\to x\leftarrow d_n$是$x$的有理数夹逼数列.在数列$\{c_n\}$中一定存在
某一项$c_N$和它后面的一切项都是正数,不然的话,如果对于
所有的$n$, 有$c_n\le 0$, 于是$\lim c_n\le 0$即$x\le 0$, 这和已知的$x>0$
矛盾.
令$c_N>0$, 则$a^{C_N}>a^0=1$, 而$a^x>a^{c_N}>1$, 这就证明了
$a^x>1$.
设$x<0$, $x=-y$, 则$y>0$, $a^x=a^{-y}=\frac{1}{a^y}$
$\because\quad a^y>1,\qquad \therefore\quad a^x=\frac{1}{a^y}<1$
$0<a<1$的情形,留给同学证明.
\end{proof}
\begin{blk}{性质2}
\begin{enumerate}
\item 若$a>1$且$x_2>x_1$, 则$a^{x_2}>a^{x_1}$
\item 若$0<a<1$且$x_2>x_1$, 则$a^{x_2}<a^{x_1}$
\end{enumerate}
\end{blk}
\begin{proof}
设$a>1$且$x_2>x_1$,
\[a^{x_2}-a^{x_1}=a^{x_1}\left(a^{x_2-x_1}-1\right)\]
因为$x_2-x_1>0$, 于是$a^{x_2-x_1}>1$, 因此$a^{x_2}-a^{x_1}>0$.
设$0<a<1$ 且$x_2>x_1$,
\[a^{x_2}-a^{x_1}=a^{x_1}\left(a^{x_2-x_1}-1\right)\]
因为$x_2-x_1>0$, 则$a^{x_2-x_1}<1$, 因此$a^{x_2}-a^{x_1}<0$.
\end{proof}
\begin{blk}{性质3}
\[\lim_{x\to 0} a^x=1\]
\end{blk}
对这个极限的证明可以仿照第一节中性质3的推论的证法
去证.利用性质3, 容易证明实指数函数处处连续.
\begin{blk}{性质4}
当$x$无限增大时,$a^x,\; (a>1)$也无限增大,可以写
成$\Lim_{x\to\infty} a^x=+\infty$ (注意这个表达式并不表示此极限存在,而是
说$a^x$可以超过任何一个指定的正数),若$0<a<1$, 则
$\Lim_{x\to\infty} a^x=0$
\end{blk}
\begin{proof}
为确定起见,设$a>1$, 令$a=1+h\; (h>0)$, 因为
\[(1+h)^n>1+nh,\qquad (n\in\mathbb{N})\]
可得:$a^n>1+n(a-1)$.
对于任意给定的一个正数$M$, 当$n>\frac{M-1}{a-1}$时,则
\[a^n>1+n(a-1)>1+\frac{M-1}{a-1}(a-1)=M\]
所以,当$x>n$时,便有$a^x>a^n>M$.
这就是说,当$x\to +\infty$时,$\lim a^x=\infty$.
设$0<a<1$, $a=\frac{1}{b}$, 则$b>1$, $a^x=\left(\frac{1}{b}\right)^n=\frac{1}{b^n}$,
任给$\varepsilon=\frac{1}{M}>0$, 则$M=\frac{1}{\varepsilon}$, 依前段,当$x>n>\frac{M-1}{a-1}$时,有$b^x>M$. 于是$a^x=\frac{1}{b^x}<\frac{1}{M}=\varepsilon$. 这就证明了当$x\to\infty$时,$\lim a^x=0,\; (0<a<1)$.
\end{proof}
\begin{blk}{性质5}
\begin{enumerate}
\item 若$a>1$, $\Lim_{x\to-\infty} a^x=0$.
\item 若$0\le a<1$, $\Lim_{x\to-\infty} a^x=+\infty$.
\end{enumerate}
\end{blk}
\begin{proof}
为确定起见,设$a>1$, 令$x=-y,\; (y>0)$,则当
$x\to-\infty$时,$y\to +\infty$, 所以
\[\Lim_{x\to-\infty} a^x=\Lim_{y\to +\infty} a^{-y}=\Lim_{y\to +\infty}\frac{1}{a^y}=\frac{1}{\Lim_{y\to +\infty}a^y}=0\]
$0<a<1$的情形,证明留给读者.
\end{proof}
总结上面的讨论,指数函数有下面的主要性质:
\begin{blk}{定理}
指数函数$f:(-\infty,+\infty)\mapsto (0,+\infty)$, 这里$f(x)=a^x,\; (a>0\text{\; 且\; }a=1)$, 满足下列三个性质:
\begin{enumerate}
\item $f(x_1+x_2)=f(x_1)f(x_2)$
\item $f(x)$是严格单调的,$a>1$时,递增;$0<a<1$时,递减
\item $f(x)$是连续的
\end{enumerate}
\end{blk}
指数函数的图象如图10.3所示:
\begin{figure}[htp]
\centering
\begin{tikzpicture}[>=latex]
\draw[->](-3.5,0)--(3.5,0)node[right]{$x$};
\draw[->](0,-1)--(0,5)node[right]{$y$};
\draw[dashed] (-3.5,1)--(2.5,1)node[right]{$y=1$};
\draw [domain=-3:3, samples=100, very thick]plot(\x, {1.6^(\x)});
\draw [domain=-3:3, samples=100, very thick]plot(\x, {0.65^(\x)});
\node at (-3,4){$y=a^x,\; (0<a<1)$};
\node at (3,4.3){$y=a^x,\; (a>1)$};
\node at (-.25,-.25){$O$};
\node at (.1,1.3)[right]{$(0,1)$};
\end{tikzpicture}
\caption{}
\end{figure}
\begin{blk}{逆定理}
任何一个满足上述性质1和2的函数
$f(x)$必定是一个指数函数,其底为$a=f(1)$.
\end{blk}
\begin{proof}
由性质1, 对于任何实数$x$, 有
\[f(0)f(x)=f(0+x)=f(x)\]
即得,$f(0)=1$. 当$f(x)$递增时,$f(1)=a>f(0)=1$, 而当
$f(x)$递减时,$f(1)=a<f(0)=1$.
由性质1:
\[\begin{split}
f(m)&=f\big((m-1)+1\big)=f(m-1)\cdot f(1)\\
&=f(m-2)\cdot \big(f(1)\big)^2=f(m-3)\cdot \big(f(1)\big)^3\\
\cdots & \cdots \cdots \cdots \cdots \\
&=\big(f(1)\big)^m=a^m
\end{split}\]
又因为
\[\begin{split}
\left[f\left(\frac{m}{n}\right)\right]^n&=f\overbrace{\left(\frac{m}{n}+\frac{m}{n}+\cdots +\frac{m}{n}\right)}^{n\text{个}}\\
&=f(m)=a^m
\end{split}\]
所以:$f\left(\frac{m}{n}\right)=\sqrt[n]{a^m}=a^{\tfrac{m}{n}}$
因为$f\left(\frac{m}{n}\right)f\left(-\frac{m}{n}\right)=f\left(\frac{m}{n}-\frac{m}{n}\right)=f(0)=1$
所以$f\left(-\frac{m}{n}\right)=\frac{1}{f\left(\frac{m}{n}\right)}=\frac{1}{a^{\tfrac{m}{n}}}=a^{-\tfrac{m}{n}}$
所以,$f(r)=a'$, 对于正、负分数$r$都成立.
再由单调性,和实指数幂的定义,就可以说明$f(x)=a^x$
对于任何实数都成立.
设$x\in\mathbb{R}$为一任意实数,$r_n\to x\leftarrow s_n$是$x$的左、右夹逼有理
数列,即
\begin{equation}
r_1\le r_2\le \cdots \le r_n\le \cdots \le x\le \cdots \le s_n\le \cdots \le s_2\le s_1
\end{equation}
并且 $\lim(s_n-r_n)=0$.
由不等式(10.2)和$f(x)$的递增性(递减性),得到
\begin{equation}
f(r_1)\le f(r_2)\le \cdots \le f(r_n)\le \cdots \le f(x)\le \cdots \le f(s_n)\le \cdots \le f(s_2)\le f(s_1)
\end{equation}
$r_n,s_n$都是有理数.
(若$f(x)$递减,我们得到不等式(10.3)的反向不等式).
不等式(10.3)可改写成
\begin{equation}
a^{r_1}\le a^{r_2}\le \cdots \le a^{r_n}\le \cdots \le a^{x}\le \cdots \le a^{s_n}\le \cdots \le a^{s_2}\le a^{s_1}
\end{equation}
而上节实数指数定义中,$a^x$是唯一能满足(10.4)的实数,所以$f(x)=a^x$.
在上面的讨论中,$x$是一个任意的实数,因此,$f(x)=
a^x$, 对于任何实数$x$恒成立.
\end{proof}
\begin{example}
设$a,b$是不等的正实数,试证
\[ a^ab^b>(ab)^{\tfrac{a+b}{2}}>a^bb^a\]
\end{example}
\begin{proof}
不妨设$a>b$, 则$\frac{a}{b}>1$, $a-b>0$.
于是,根据实指数幂性质1,可得:
\[\frac{a^ab^b}{(ab)^{\tfrac{a+b}{2}}}=a^{\tfrac{a-b}{2}}\cdot b^{\tfrac{b-a}{2}}=\left(\frac{a}{b}\right)^{\tfrac{a-b}{2}}>1\]
由于$a>0,\; b>0\Rightarrow ab>0$, 因此,$(ab)^{\tfrac{a+b}{2}}>0$, 所
以,有
\[ a^ab^b>(ab)^{\tfrac{a+b}{2}}\]
另外,根据同样的道理,有
\[\frac{(ab)^{\tfrac{a+b}{2}}}{a^bb^a}=a^{\tfrac{a-b}{2}}\cdot b^{\tfrac{b-a}{2}}=\left(\frac{a}{b}\right)^{\tfrac{a-b}{2}}>1\]
又$a^b>0$, $b^a>0$. 所以$(ab)^{\tfrac{a+b}{2}}>a^b b^a$, 这就证明了
\[a^ab^b>(ab)^{\tfrac{a+b}{2}}>a^b b^a\]
\end{proof}
\section*{习题10.2}
\addcontentsline{toc}{subsection}{习题10.2}
\begin{enumerate}
\item 利用实指数幂的性质,指出下列不等式中,$a$是大
于1, 还是大于0而小于1?
\begin{multicols}{2}
\begin{enumerate}
\item $a^{\sqrt{2}}<a^{\tfrac{\sqrt{2}}{2}}$
\item $a^{-\sqrt{3}}>a^2$
\item $a^{-\sqrt{5}-\sqrt{7}}>a^{-5}$
\item $a^{1+\sqrt{5}}<a^{2+\sqrt{2}}$
\item $a^{\sqrt{7}+\sqrt{2}}<a^{\sqrt{6}+\sqrt{3}}$
\end{enumerate}
\end{multicols}
\item 作下列各函数的图象:
\begin{multicols}{2}
\begin{enumerate}
\item $y=3^x$
\item $y=3^{-x}$
\end{enumerate}
\end{multicols}
\item \begin{enumerate}
\item 证明$f(x)=\frac{2^x+2^{-x}}{2}$是偶函数,并作出它的图象;
\item 当$x$为何值时,$f(x)$有最小值,并求最小值.
\end{enumerate}
\item 设$a,b,c$是不等的正数,证明:
\begin{enumerate}
\item $a^{2 a} b^{2 b} c^{2 c}>a^{b+c} b^{c+a} c^{a+b}$
\item $a^{a} b^{b} c^{c}>(a b c)^{\tfrac{a+b+c}{3}}$
\end{enumerate}
(提示:利用例10.2的结果)
\item 证明:
\begin{enumerate}
\item 当$n$是1或不小于5的自然数时,总有$2^n>n^2$;
\item $\Lim_{n\to\infty}\frac{2^n}{n}=\infty$.
\end{enumerate}
\end{enumerate}
\section{对数函数}
由实数幂的定义,我们得知指数函数
\[a^x,\quad (a>0,\;a\ne 1),\qquad x\in\mathbb{R}\]
的值都是正的,现在还要进一步说明指数函数的值域是正实
数集,也就是必须证明下面的命题.
\begin{blk}{命题}
给定不等于1的正实数$a$, 对于任意正数$b$, 一
定存在唯一的一个实数$c$, 满足下列方程
$$a^c=b$$
\end{blk}
\begin{proof}
为确定起见,设$a>1$, 依实指数函数的性质5,
$\Lim_{x\to-\infty}a^x=0$, 可以找出这样一数$c_1$以使$a^{c_1}<b$, 依$a^x,\;(a>1)$
是增函数且$\Lim_{x\to+\infty}a^x=+\infty$, 可以找出这样的数$c_2>c_1$, 以使
$a^{c_2}>b$, 现在由连续函数中间值定理知道,在$c_1$与$c_2$之间有
实数$c$以使$a^c=b$, 再由单调性知道这个数是唯一的.类似
地可以证明$0<a<1$的场合,这个证明留给同学补全.
\end{proof}
现在我们根据第八章第五节反函数定理可以说由指数函数
得到一个定义在正实数变域上的反函数,称为对数函数,记
作$f:\mathbb{R}^+\mapsto \mathbb{R}$, 这里$f(x)=\log_a x$, 它是连续的单调函数.正式定义如下:
\begin{blk}{定义}
若$a>0$且$a\ne 1$, 那么$y=\log_a x$, 当且仅当
$x=a^y$. 我们称$y$是以$a$为底的对数,函数$f:\mathbb{R}^+\mapsto \mathbb{R}$,这里
$f(x)=\log_a x$称为\textbf{对数函数}.
\end{blk}
这个定义导致下面有用的结果:
\begin{enumerate}
\item $a^{\log_a x}=x,\qquad \log_a a^x=x$
\item $\log_a (xy)=1ogax+\log_a y$
\item $\log_a \frac{x}{y}=\log_a x-\log_a y$
\item $\log_a x^r=r\log_a x$
\item $\log_a b=\frac{\log_c b}{\log_c a}$(换底公式).
\end{enumerate}
证明过程请看第三册第一章.
从函数的图象来说:$y=\log_a x,\; (x\in\mathbb{R})$的图象能由$y=a^x,\; (x\in\mathbb{R})$的图象经$y=x$的反射而得到,如图10.4.
\begin{figure}[htp]
\centering
\begin{tikzpicture}[>=latex]
\begin{scope}
\draw[->](-2,0)--(3.5,0)node[right]{$x$};
\draw[->](0,-2)--(0,3.5)node[right]{$y$};
\draw[thick] (-1.5,-1.5)--(3,3)node[right]{$y=x$};
\draw[domain=-1.5:2, samples=100, very thick]plot(\x,{1.8^(\x)});
\draw[domain=-1.5:2, samples=100, very thick]plot({1.8^(\x)},\x);
\node at (2,3.5){$y=a^x,\; (a>1)$};
\node at (3.2,2)[right]{$y=\log_a x,\; (a>1)$};
\node at (1,0)[below]{1};
\node at (0,1)[left]{1};
\node at (.25,-.25){$O$};
\end{scope}
\begin{scope}[yshift=-6cm]
\draw[->](-2.5,0)--(3.5,0)node[right]{$x$};
\draw[->](0,-3)--(0,3.5)node[right]{$y$};
\draw[thick] (-2,-2)node[below]{$y=x$}--(3,3);
\draw[domain=-2:3, samples=100, very thick]plot(\x,{0.6^(\x)});
\draw[domain=-2:3, samples=100, very thick]plot({0.6^(\x)},\x);
\node at (-2,3){$y=a^x,\; (0<a<1)$};
\node at (3,-2)[right]{$y=\log_a x,\; (0<a<1)$};
\node at (1,0)[below]{1};
\node at (0,1)[left]{1};
\node at (.25,-.25){$O$};
\end{scope}
\end{tikzpicture}
\caption{}
\end{figure}
由于$x$轴是指数函数图象的渐近线,故$y$轴是对数函数
图象的渐近线.相应于指数函数的极限值:
\[\begin{split}
\lim_{x\to -\infty} a^x=0&,\qquad a>1\\
\lim_{x\to +\infty} a^x=0&,\qquad 0<a<1
\end{split}\]
有对数函数的极限值:
\[\lim_{x\to 0^+} \log_a x=\begin{cases}
-\infty, & a>1\\
+\infty ,& 0<a<1
\end{cases}\]
相应于指数函数的特征性质,也就有对数函数的特征
性质:
\begin{enumerate}
\item $\log_a (x_1\cdot x_2)=\log_a x_1+\log_a x_2$
\item 单调性.当$a>1$递增;当$0<a<1$递减.
\item 连续性.即在$(0,+\infty)$内处处连续.同样地,对
应于第三节中的定理,总结成下面的定理.
\end{enumerate}
\begin{blk}{定理}
对数函数$f(x)=\log_a x$满足下列性质:
\begin{enumerate}
\item $f(x_1\cdot x_2)=f(x_1)+f(x_2)$
\item 单调性.$a>1$时,递增;$0<a<1$时,递减.
\item 在$x>0$半直线上,处处连续.
\end{enumerate}
\end{blk}
\begin{blk}{逆定理}
任何一个满足性质1、2的函数$f(x)$一
定是一个对数函数,即存在适当的$a$, 使得$f(x)=\log_a x$.
\end{blk}
\begin{proof}
由性质1,$f(x_1)=f(x_1\cdot 1)=f(x_1)+f(1)$.
因此$f(1)=f(x_1)-f(x_2)=0$.
我们先任取一常数$A>1$, 则由性质2知
\[f(A)\ne f(1)=0\]
再由性质1
\[f(A^m)=\underbrace{f(A)+f(A)+\cdots+f(A)}_{\text{$m$项}}=mf(A),\qquad m\in\mathbb{N}\]
又
\[\begin{split}
mf(A)&=f(A^m)=f\left(\left((A)^{\tfrac{m}{n}}\right)^n\right)\\
&=\underbrace{f\left(A^{\tfrac{m}{n}}\right)+\cdots+f\left(A^{\tfrac{m}{n}}\right)}_{\text{$n$项}}=nf\left(A^{\tfrac{m}{n}}\right)
\end{split} \]
$\therefore\quad f\left(A^{\tfrac{m}{n}}\right)=\frac{m}{n}f(A),\qquad m,n\in\mathbb{N}$
又$\because\quad f\left(A^{\tfrac{m}{n}}\right)+f\left(A^{-\tfrac{m}{n}}\right)=f\left(A^{\tfrac{m}{n}}\cdot A^{-\tfrac{m}{n}}\right)=f(A^0)=f(1)=0$
$\therefore\quad f\left(-A^{\tfrac{m}{n}}\right)=-f\left(A^{\tfrac{m}{n}}\right)=-\frac{m}{n}f(A)$
综合上面所证,所以对于所有有理数$r\in\mathbb{Q}$, 都有
\[f(A^r)=rf(A)\]
从此不难用单调性和极限过程,导出
\begin{equation}
f(A^{\beta })=\beta f(A),\qquad \beta \in\mathbb{R}
\end{equation}
令$A^{\beta }=x$, 则$\beta =\log_A x$, 于是(10.5)可写成
\begin{equation}
f(x)=f(A)\cdot \log_A x
\end{equation}
这样我们得到一个连续的单调的对数型函数(10.6). 为了化去
常数因子$f(A)$, 我们要用一些技巧如下:令$a=A^{\tfrac{1}{f(A)}}$
, 于是
\[1=\log_a a=\log_a A^{\tfrac{1}{f(A)}}=\frac{1}{f(A)}\cdot \log_a A\]
即:$f(A)=\log_a A$, 代入(10.6), 得到:
\[f(x)=\log_A x\cdot \log_a A=\log_a x\]
\end{proof}
\section*{习题10.3}
\addcontentsline{toc}{subsection}{习题10.3}
\begin{enumerate}
\item 求下列各函数的定义域与值域,如果它们是可逆的,
写出以$x$为自变数的反函数.
\begin{multicols}{2}
\begin{enumerate}
\item $y=\log_2(x-2)$
\item $y=\log_2\frac{1}{x}$
\item $y=e^{-x}$
\item $y=\sqrt{\lg\cos2\pi x}$
\end{enumerate}
\end{multicols}
\item 计算下列各式的值:
\begin{multicols}{2}
\begin{enumerate}
\item $2^{\log_4 9}$
\item $5^{\log_{0.2} 7}$
\item $3^{\log_{\sqrt{2}}6}$
\item $6^{1+\log_6 5}$
\item $25^{\tfrac{1}{3} \log 5^{27}-\log_{5} 4}$
\item $10^{\lg\sqrt{100}}$
\item $\log _{\sqrt{4-\sqrt{15}}} \sqrt{4+\sqrt{15}}$
\item $4-\lg 8-3 \lg 5$
\item $ \lg ^{2} 5+\lg 2 \lg 50$
\item $\frac{3 \lg 1728}{1+\frac{1}{2} \lg 0.36+\frac{1}{3} \lg 8}$
\item $\log _{a} b \cdot \log _{b} c \cdot \log _{c} d$
\item $\left(\log _{2} 3+\log _{4} 9\right)\left(\log _{3} 4+\log _{9} 2\right)$
\end{enumerate}
\end{multicols}
\item 证明下面的恒等式:
\begin{enumerate}
\begin{multicols}{2}
\item $\log_a b=\frac{\log_c b}{\log_c a}$
\item $\log_a b\cdot \log_b a=1$
\item $\frac{\log_a x}{\log_{ab} x}=1+\log_a b$
\end{multicols}
\item \[\begin{split}
&\quad \frac{1}{(\log_x 2)(\log_x 4)}+ \frac{1}{(\log_x 4)(\log_x 8)}+ \frac{1}{(\log_x 8)(\log_x 16)}+\cdots \\
&+ \frac{1}{(\log_x 2^{n-1})(\log_x 2^n)}=\left(1-\frac{1}{n}\right)\left(\frac{1}{\log_x 2}\right)^2
\end{split}\]
\end{enumerate}
\item \begin{enumerate}
\item 已知$\lg2=a$, $\lg7=b$, 求$\log_8 9.8$;
\item 已知$\log_{18}9=a$, $\log_{18}5=b$, 求$\log_{36}45$;
\item 已知$\lg198=2.2966$, $\lg2=0.3010$, $\lg3=0.4771$, 求$\lg 11$;
\item 已知$\log_{12}7=m$, $\log_{12}3=n$, 求$\log_{18}63$.
\end{enumerate}
\item 已知$\lg3=0.4771$, 问$\left(\frac{1}{3}\right)^{20}$
表成小数时,不等于0
的第一个有效数字出现在哪里?
\item 证明下面不等式:
\begin{enumerate}
\item $\left|\log _{a} b\right|+\left|\log _{b} a\right| \geqslant 2$
\item $\frac{1}{\log _{2} \pi}+\frac{1}{\log _{\pi} 2}>2$
\item 若 $a>b>0$ 且 $c>1$, 则 $\log _{c} \frac{b}{a}<\log _{c} \frac{1+b}{1+a}$.
\item 若 $t>-1$, $\varphi(t)=-\lg(1+t)$, 则 $\varphi\left(\frac{t_{1}+t_{2}}{3}\right)<\frac{\varphi\left(t_{1}\right)+\varphi\left(t_{2}\right)}{2}$
\end{enumerate}
\item 当 $2 x+5 y=20$ 时, 求 $\log _{2} x+\log _{2} y$ 的最大值.
\item 设 $x>1$, $y>1$且 $2 \log _{x} y-2 \log _{y} x+3=0$, 那么 $x^{2}-$ $4 y^{2}$ 的最小值是多少?
\item 设 $x>2$, $y>2$, 比较下列各式的大小:
$$\log _{2} \frac{x+y}{2} ;\qquad \frac{1}{2} \log _{2}(x+y); \qquad \frac{1}{2}\left(\log _{2} x+\log _{2} y\right)$$
\item 求证等比数列的各项的对数组成等差数列.
\item 有等比数列, 它的公比为 2, 项数为10, 如果各项 取以2 为底的对数, 它们的和是25, 求这等比数列的和.
\item 试问数列
$$\lg100,\; \lg\left(100\sin\frac{\pi}{4}\right),\; \lg\left(100\sin^2\frac{\pi}{4}\right),\; \ldots ,\; \lg\left(100\sin^{n-1}\frac{\pi}{4}\right),\; \ldots$$
的前多少项的和的值最大?并求
出最大值(这里取$\lg2=0.3010$).
\end{enumerate}
\section{指数方程与对数方程}
指数中含有未知数的方程叫做\textbf{指数方程}.下面我们介绍
几种常见的指数方程及其解法.
\subsection{可化为$\alpha^{f(x)}=\alpha^{g(x)}\; (a>0\text{\;且\;}a\ne 1)$的指数方程}
对于这类方程,我们根据指数函数的单调性得到$\alpha^{f(x)}=\alpha^{g(x)}$成立的必要充分条件是$f(x)=g(x)$. 因此,指数方程$\alpha^{f(x)}=\alpha^{g(x)}$在$a>0$且$a\ne 1$的条件下就可以转化为代数方程$f(x)=g(x)$来解.
\begin{example}
解方程$5^{-x}\cdot 50^x=\frac{1}{1000(10^{2x-1})^{-3}}$
\end{example}
\begin{solution}
原方程化简为 $(5^{-1}\cdot 50)^x=\frac{10^{6x-3}}{10^3}$,
即:
\[10^x=10^{6x-6}\]
由于底数$a=10>0$且$\ne 1$, 得到
\[x=5x-6 \quad \Rightarrow\quad x=\frac{6}{5}\]
所以原方程的解集是$\left\{\frac{6}{5}\right\}$.
\end{solution}
\begin{example}
解方程$17^{3x^2+x-2}=1$
\end{example}
\begin{solution}
$\because\quad 1=17^0$,原方程可写成
$$17^{3x^2+x-2}=17^{0}$$
于是根据指数函数的单调性,得到
\[3x^2+x-2=0\]
由此
\[x_1=\frac{2}{3},\qquad x_2=-1\]
所以 原方程的解集是$\left\{-1,\frac{2}{3}\right\}$.
\end{solution}
\subsection{可化为形如$a^{f(x)}=b^{g(x)}$的指数方程}
这里($a>0$, $b>0$, $a\ne 1$, $b\ne 1$),一般用两边取对数的方法来解.
\begin{example}
解方程$17^x=300$
\end{example}
\begin{solution}
两边取常用对数,得到
\[\begin{split}
x\lg17&=\lg300\\
x&=\frac{\lg300}{\lg 17}\approx \frac{2.4771}{1.2304}\approx 2.0132
\end{split}\]
\end{solution}
\begin{example}
解方程$5^{2x}-7x-35\cdot 5^{2x}+35\cdot 7^x=0$
\end{example}
\begin{solution}
原方程化简为
$7^x (35-1)=5^{2x}(35-1)$
两边除以34, 得到:$5^{2x}=7^x$
两边取常用对数
\[\begin{split}
2x\lg5&=x\lg7\\
x(2\lg5-\lg7)&=0
\end{split} \]
$\because\quad 2\lg5-\lg7=\lg25-\lg7\ne 0,\qquad \therefore\quad x=0$
因此,原方程的解集是$\{0\}$.
\end{solution}
\subsection{可化为一元二次方程的指数方程}
\begin{example}
解方程$\left(\sqrt{2-\sqrt{3}}\right)^x+\left(\sqrt{2+\sqrt{3}}\right)^x=4$
\end{example}
\begin{solution}
注意到$\sqrt{2-\sqrt{3}}\cdot \sqrt{2+\sqrt{3}}=\sqrt{4-3}=1$,
原方程的两边乘以$\left(\sqrt{2-\sqrt{3}}\right)^x$, 得到
\[\left(\sqrt{2-\sqrt{3}}\right)^{2x}+1=4\left(\sqrt{2-\sqrt{3}}\right)^x\]
即
$\left(\sqrt{2-\sqrt{3}}\right)^{2x}-4\left(\sqrt{2-\sqrt{3}}\right)^x+1=0$
$\therefore\quad \left(\sqrt{2-\sqrt{3}}\right)^x=2+\sqrt{3}\quad \text{或}\quad \left(\sqrt{2-\sqrt{3}}\right)^x=2-\sqrt{3}$
即:
\[\left({2-\sqrt{3}}\right)^{\tfrac{x}{2}}=\left({2-\sqrt{3}}\right)^{-1}\quad \text{或}\quad \left({2-\sqrt{3}}\right)^{\tfrac{x}{2}}=2-\sqrt{3}\]
$\therefore\quad x=-2\quad \text{或}\quad x=2$
$\therefore\quad $原方程的解集是$\{-2,2\}$.
\end{solution}
未知数前面有对数符号的方程称为对数方程.解对数方
程一般常用的方法是根据对数定义直接把对数式的等式写成
指数形式的等式.也有时根据对数函数的单调性把对数方程
化为代数方程来解.但是必须注意在解对数方程之前,应该
先确定使方程中的对数都有意义的定义域,由此便确定了方
程的根的上、下界.在求得对数方程之解后,应该舍去在根
的上、下界之外的增根,换言之,把那些使真数或底数为非
正数或使底数等于1的根舍去,下面介绍几种常见的对数
方程.
\subsection{形如$\log_{f(x)}g(x)=c$\; (其中$c$是常数)的对数方程}
可以根据对数定义将它化为指数形式的等式去解.
\begin{example}
解方程$\log_{x-5}(3x^2-16x+29)=2$
\end{example}
\begin{solution}
方程中的对数有意义必须
\[\begin{cases}
x>5\quad\text{且}\quad x-5\ne 1 \\
13x^2-16x+29>0
\end{cases}\]
根据对数定义得到
\[ 3x^2-16x+29=(x-5)^2\]
解得:$x_1=1,\qquad x_2=2$.
由于1和2都小于5, 所以原方程没有解,即原方程的
解集是空集.
\end{solution}
\begin{example}
解方程$\log_3[3+2\lg(1+x)]=0$
\end{example}
\begin{solution}
根据对数定义得到$3+2\lg(1+x)=1$,
即:
\[ \lg(1+x)=-1\]
再由对数定义有
\[\begin{split}
1+x&=10^{-1}\\